Volume Generated by Rotating Two Curves About x-Axis

  • Thread starter Thread starter jack1234
  • Start date Start date
  • Tags Tags
    Volume
Click For Summary
The discussion revolves around calculating the volume generated by rotating the area between the curves y=e^x and y=1+2e^(-x) about the x-axis. Initially, there was confusion regarding the correct formulation of the curves, particularly concerning their behavior at infinity. It was clarified that the curves intersect at the point (log 2, 2), establishing a finite region for volume calculation. The bounded area is symmetric about the y-axis, allowing for the use of integration to determine the volume. The conversation emphasizes the importance of correctly identifying the curves and their intersection points for accurate volume computation.
jack1234
Messages
132
Reaction score
0
A question here:
Given two curves
y=e^x
y=1+2e^(-x)

The region in the first quadrant that is bounded by the y-axis and these two curves is rotated through one complete revolution about the x-axis. Calculate the exact volume of the solid generated.

My problem is, in the first quadrant, y=1+2e^(-x) seems touching the x-axis at x=\infty, so how do we find the volume?
 
Last edited:
Physics news on Phys.org
You sure you copied the problem correctly?
 
Thanks, you are right that the question has not been copied correctly. I have changed
y=1+2e^(x)
to
y=1+2e^(-x)
Please refer to the original question again. Very sorry for any inconvenience caused.
 
The two curves cross at (0,3), of course, and the region under the two curves is symmetric about the y-axis. However, if that really is the correct formula, because y goes to 1 as x goes to \infty, and as x goes to -\infty, the volume generated contains an infinitely long cylinder of radius 1 and so is not finite.
 
A start

The two curves are y=e^{x} and y=1+2e^{-x} which intersect when e^{x}=1+2e^{-x} multiply by e^x to get e^{2x}-e^{x}-2=0 so by the quadratic formula we have e^{x}=2 or x = \log {2} so the curves meet at the point (log 2, 2). The other boundary is the y-axis so the bounded area is now finite (see attached plot) and to be rotated about the x-axis, so do an integral :smile: . --Ben
 

Attachments

  • exp(x),1+2exp(-x),plot.gif
    exp(x),1+2exp(-x),plot.gif
    1 KB · Views: 473
Question: A clock's minute hand has length 4 and its hour hand has length 3. What is the distance between the tips at the moment when it is increasing most rapidly?(Putnam Exam Question) Answer: Making assumption that both the hands moves at constant angular velocities, the answer is ## \sqrt{7} .## But don't you think this assumption is somewhat doubtful and wrong?

Similar threads

  • · Replies 1 ·
Replies
1
Views
2K
Replies
3
Views
2K
  • · Replies 3 ·
Replies
3
Views
3K
Replies
2
Views
2K
  • · Replies 5 ·
Replies
5
Views
5K
Replies
1
Views
1K
  • · Replies 6 ·
Replies
6
Views
2K
  • · Replies 4 ·
Replies
4
Views
1K
  • · Replies 8 ·
Replies
8
Views
2K
  • · Replies 10 ·
Replies
10
Views
4K